Manipal Medical Manipal Medical Solved Paper-2013

  • question_answer
    Figure shows three points A, B and C in a region of uniform electric field E. The line AB is perpendicular and BC is parallel to the field lines. Then which of the following holds good?

    A)  \[{{V}_{A}}={{V}_{B}}={{V}_{C}}\]         

    B)  \[{{V}_{A}}={{V}_{B}}>{{V}_{C}}\]

    C)  \[{{V}_{A}}={{V}_{B}}<{{V}_{C}}\]          

    D)  \[{{V}_{A}}>{{V}_{B}}={{V}_{C}}\]

    Correct Answer: B

    Solution :

    Electric line of forces constitute electric field. In an electric field electric line of force always flow form higher potential to lower potential. Hence, A and B are at same higher potential and B is at lower potential. Thus \[{{V}_{A}}={{V}_{B}}>{{V}_{C}}\] where\[{{V}_{A}},{{V}_{B}}\]and\[{{V}_{C}}\]represent the electric potential at the points A, B and C respectively.


You need to login to perform this action.
You will be redirected in 3 sec spinner